You are on page 1of 19

CICLO PREUNIVERSITARIO ADMISIN 2007-II

FSICA

A) VFV
D) FFF

1. La ... es aquella capaz de


cambiar el estado de movimiento de
una partcula, y su expresin
dimensional es
Completan la oracin:
A) masa ; M
B) aceleracin; LT2
C) fuerza ; MLT2
D) magia ; MLT
E) imaginacin; *
2. Con
relacin
a
las
siguientes
proposiciones sobre la primera Ley de
Newton indique verdadero (V) o falso
(F):
I. Una partcula que se encuentra
viajando con velocidad constante
continuar en dicho estado de
movimiento, salvo que una fuerza
actu sobre l.
II. Un cuerpo que est en reposo
puede iniciar su movimiento
repentinamente sin la accin de
una fuerza.
III. Para que un cuerpo tenga un
movimiento rectilneo uniforme, es
necesario la accin de una fuerza
constante.
A) VVV
D) FVV

B) VVF
E) VFF

C) VFV

3. Respecto a la 1 Ley de Newton seale


la veracidad (V) o falsedad (F) de las
siguientes proposiciones.
I. La primera ley de Newton es
aplicable tambin en un sistema de
referencia acelerado respecto a
Tierra.
II. En un reposo instantneo respecto
a Tierra es aplicable la 1 Ley de
Newton.
III. Los objetos en reposo o con MRU
respecto a Tierra cumplen con la
1 Ley de Newton.
CEPRE-UNI

SEMINARIO N 02

B) VVF
E) FVF

C) FFV

4. Se propone lo siguiente:
I. La expresin dimensional MLT2
tiene unidad newton en el SI.
II. La 1era Ley de Newton explica
porqu al romperse la cuerda que
ata a una piedra en movimiento
circular, sta se mueve en una
recta tangente a la trayectoria que
tena.
III. Si un objeto se mueve con MRU
respecto de otro, entonces segn
la 1era Ley de Newton existe
equivalencia entre ambos, es decir
est en equilibrio uno respecto al
otro.
Son correctas:
A) I
D) I y II

B) II
E) II y III

C) III

5. Respecto a la 1era Ley de Newton,


determinar
si las proposiciones
siguientes son falsas (F) o verdaderas
(V).
I. Si la fuerza sobre una partcula es
nula, sta necesariamente tiene un
MRU.
II. Si la fuerza sobre una partcula es
nula, sta necesariamente est en
reposo.
III. Si no existe accin externa sobre
una partcula, sta necesariamente
est en reposo o con MRU.
A) VVV
D) FVV

B) FFF
E) VFF

C) FFV

6. Indique verdadero (V) o falso (F), segn


corresponda:
I. Cuando se lanza una moneda
hacia arriba, en el punto ms alto
de su trayectoria, se encuentra en
equilibrio.
II. Un tren se mueve con
ur velocidad
constante igual a V 0 . Si una
FSICA

CICLO PREUNIVERSITARIO ADMISIN 2007-II

manzana se encuentra en reposo


sobre una mesa del tren entonces
sobre la manzanaurdebe actuar una
fuerza paralela a V 0 .
III. La interaccin entre un cuerpo y la
superficie que le sirve de apoyo es
de naturaleza electromagntica.
A) VVV
D) VFV

B) VVF
E) FFV

A) VVV
D) FVF

B) FFF
E) VVF

B) VVF
E) FFF

y
x

Cuerda

A)

B)

D)

C)

E)

C) FFV

8. Seale verdadero (V) o falso (F) las


siguientes proposiciones con respecto
a la 3ra Ley de Newton.
I. Es consecuencia de la primera ley
de Newton.
II. Las fuerzas se presentan en pares.
III. Es vlida solamente usando los
cuerpos estn adyacentes.
A) VVV
D) FVF

C) VFF

7. Indicar la veracidad (V) o falsedad (F)


de las proposiciones siguientes:
I. Las fuerzas bsicas en la
naturaleza son cinco.
II. Las
fuerzas
de
interaccin
electrodbil tienen menor alcance
que las de interaccin gravitatoria.
III. El cambio en el estado de
movimiento de los electrones en
torno del ncleo de un tomo, es el
resultado de la interaccin nuclear
fuerte.

SEMINARIO N 02

C) VFF

10. La figura muestra a Pedro (peso P)


sentado en reposo sobre un andamio
(peso p), tirando de una cuerda ideal
(que pasa por la polea, tambin ideal)
que soporta una tensin T. cul es el
DCL correcto?

9. Las esferas A y B estn apoyadas


sobre superficies lisas tal como lo
indica la figura. Indique el D.C.L. de la
esfera A, si sobre B acta una fuerza

vertical F F $j

CEPRE-UNI

FSICA

CICLO PREUNIVERSITARIO ADMISIN 2007-II

A)

B)

SEMINARIO N 02

Reaccin de
la pared

C)

peso

p
T

D)

2T

A) Todas
D) I y III
P
Ptp

E)

2T

Reaccin
normal del
piso

B) Solo I
E) Ninguna

Rozamiento
B

C) I y II

12. Dos cilindros macizos de


y 15 kg
de masa, se apoyan sin rozamiento
como se muestra en la figura. Calcule
el ngulo que forma con la horizontal
la lnea que une los centros de los
cilindros.
liso
?

P+p

11.La barra que se muestra es


homognea y se mantiene en
equilibrio apoyada en una pared
vertical lisa y en un piso horizontal
rugoso como se indica en el dibujo (A).
es correcto que:
I. Al descomponer la reaccin del
piso sobre la barra. Su diagrama
de cuerpo libre queda como indica
la figura (B).
II. El peso y la reaccin normal del
piso forman un par de accinreaccin.
III. La reaccin de la pared y el
rozamiento forman un par de
accin-reaccin.

53

37

A) arctg(1/4)
C) 30
E) 60

B) arctg(1/2)
D) 45

13. En la figura halle el valor de la normal


N en funcin de W considerando que
el sistema est en equilibrio.

u
r
T
30
W
30

A) W
C) W 3 / 3
E) 3W
CEPRE-UNI

B) W 2 / 3
D) 2W

FSICA

CICLO PREUNIVERSITARIO ADMISIN 2007-II

14. En la figura las masas estn en


equilibrio. Seale la veracidad (V) o
falsedad (F) de las siguientes
proposiciones:
m
1
I.
m'
II. La normal N sobre m' en funcin
de vara aproximadamente
segn la figura.

16. Una barra homognea de


N de
peso y 10 m de longitud puede girar
alrededor del punto O, si es
abandonada en la posicin A.
Determine el torque (en N.m) debido al
peso, en el instante que la barra pasa
por B, con respecto del centro de
rotacin.
A

III.

SEMINARIO N 02

B
x

m'
1
m

60

m'

A) FFF
D) FVF

B) VVF
E) FVV

C) VFF

15. Dos cilindros de radios R1 4m y


R2

r
g

masas

m1 10 kg

m2 2kg
respectivamente
se
encuentran en equilibrio. Halle la
constante K en N/m del resorte si ste
se deforma 8 cm.

A) 20 3 $
k

B) 50 3 $
k

C) - 20 3 $
k
E) - 50 3 $
k

D) - 25 3 $
k

17. Sobre una placa actan tres fuerzas


como se muestran cuyos mdulos son:
F1 60 N , F2 50 N , F3 10 N . Si P
es punto medio, determine el torque
resultante (en N.m) respecto al punto
P.
r
r
F2
F1
P

53

$
j
3m

$
i
4m

A) 34 k$
D) 42 k$

37

A) 150
D) 600

CEPRE-UNI

B) 300
E) 900

B) 38 k$
E) 32 k$

r
F3
C) 26 k$

C) 450

FSICA

CICLO PREUNIVERSITARIO ADMISIN 2007-II

18. Determine
el torque (en N m ) de la
r
fuerza F respecto del punto A.

SEMINARIO N 02

r 45
v
0,5

z
5

3m

r
100 N

y(m)

A
(4,6,0)

C
(8,3,0)

x(m)

A) 148,5i$ 124 $
j 248,6k$
B) 184,6i$ 248 $
j 424,8k$
C) 151,5i$ 202 $
j 363,6k$
D) 38,5i$ 145,6 $
j 228,6k$

A) 80
D) 60

B) 100
E) 110

C) 70

21. En la figura la viga de 1 m con una


masa de 6 kg est apoyada en un piso
rugoso y en una esfera lisa como se
indica. Si la esfera de radio a tiene una
masa de 10 kg. Determine (en N) la
reaccin normal del piso sobre la viga
cuando sta se encuentra a punto de
deslizar.

E) 148,6i$ 264,8 $
j 326,8k$
19. En la figura calcule la fuerza de
friccin. Considere que la barra es
homognea y se encuentra en
equilibrio.

A) 25
D) 45

T 30
W = 200 N

30

A) 25
D) 50

B) 25 3
E) 50 3

C) 50

20. Una bolita de peso P parte de A y se


mueve hacia B con una velocidad
constante de
cm/s, sobre una viga
horizontal. Considerando el peso de la
viga despreciable, determine el tiempo
en segundos en el cual la reaccin en
el soporte A es el doble de la reaccin
en B.

CEPRE-UNI

s 0,5

45
B) 30
E) 50

C) 40

22. La barra homognea de


N de
peso, se mantiene en reposo en la
posicin mostrada, halle la tensin en
la cuerda (en N).
A)
B)
C)
D)
E)

4
5
6
8
10

37

FSICA

CICLO PREUNIVERSITARIO ADMISIN 2007-II

23. En la figura se muestran 2 esferas del


mismo material cuyos radios son
a cm y b 2 cm apoyadas en
una superficie hemisfrica de radio 11
1
cm si ignoramos la friccin y sen ,
6
determine que define la posicin de
equilibrio.

SEMINARIO N 02

2
W
5
3
W
D)
20
A)

9
W
100
3
W
E)
100
B)

C)

3
W
25

26. El sistema es ideal, las poleas no tiene


pesos, no hay friccin, determine la
razn de masas mB /mA , para obtener
el equilibrio.

A) 15
D) 45

B) 30
b
E) 22,5

C) 37

37
B

24. Se dispone de una varilla de 3 m para


levantar un saco de
kg. Si se
utiliza una fuerza de 100 N, cul es la
mxima distancia (en cm), medida
desde el extremo donde se coloca al
saco, donde debe ubicarse el apoyo a
la varilla?
A) 15
B) 27
C) 32
D) 45
E) 52
25. Determine la fuerza F necesaria para
mantener al bloque de peso W en
equilibrio, sobre el plano inclinado liso.
Todas las poleas son de peso
despreciable.
37
F

A) 3,0
D) 1,2

B) 2,4
E) 0,4

C) 1,8

27. Un bloque de
kg se encuentra
sobre un plano inclinado 45, si la
fuerza de rozamiento entre el bloque y
el plano es insignificante, determine la
fuerza mnima horizontal (en N) capaz
de mantener al bloque en reposo
g 10 m/s2
A) 500
C) 1000
E) 750

B) 500 2
D) 1000 2

28. Entre que valores, debe estar la fuerza


que la persona aplica a la caja
(paralela al plano inclinado) sin que
esta llegue a deslizar. (Peso de la caja
100 N; s 3/6, 30 ).

37

CEPRE-UNI

A) 20 y 50
D) 25 y 80

B) 30 y 90
E) 30 y 75
FSICA

C) 25 y 75
6

CICLO PREUNIVERSITARIO ADMISIN 2007-II

29. Un bloque de
kg de masa est
apoyado
sobre
una
superficie
horizontal spera, al tomar mediciones
de la fuerza horizontal F y la fuerza de
rozamiento se obtiene la siguiente
grfica. Indique las afirmaciones
verdaderas.
m

SEMINARIO N 02

31. En la figura la esfera en equilibrio est


apoyada en una rampa rugosa.
Calcule la fuerza de friccin si la
prolongacin de la cuerda pasa por el
centro de dicha esfera.
C

f(N)

A) W cos
C) 0
E) W ctan

600
400
200
0 200 400 600 800

F(N)

I. El coeficiente de rozamiento
esttico entre las superficies es
0,75.
II. El coeficiente de rozamiento por
deslizamiento es 0,5.
III. La fuerza de rozamiento por
deslizamiento es 800 N.
A) Solo I
D) Solo I y II

B) W sen
D) Wtan

32. El muchacho desea poner en


movimiento un bloque cuyo peso es 4
veces el suyo. Halle el mnimo
coeficiente de rozamiento que debe
existir entre sus zapatos y el piso.
Entre el bloque y el piso: s 0,12 ;
k 0,10 .

B) Solo II
C) Solo III
E) Solo II y III

30. Del grfico, al aplicar una fuerza


F = 60 $
kg de
i N, el bloque de
masa, se encuentra en estado de
movimiento inminente hacia arriba; si
la esfera tiene una masa de 5 kg,
determine el coeficiente de rozamiento
esttico entre el bloque y la
pared g 10 m/s2 .
37

A) 0,03
D) 0,40

B) 0,06
E) 0,48

C) 0,12

33. La figura muestra un bloque de


15 3 kg , al cual se le aplica una
fuerza horizontal de
N, haciendo
que el bloque se mueva con velocidad
constante a lo largo del plano,
determine la magnitud (en N) de la
reaccin del plano inclinado sobre el
bloque. g 10 m/s2 .
F

F
A) 0,5
D) 0,2
CEPRE-UNI

B) 0,4
E) 0,6

C) 0,3

A) 150
D) 300 3

B) 150 3
E) 450
FSICA

C) 300
7

CICLO PREUNIVERSITARIO ADMISIN 2007-II

34. Mediante una fuerza horizontal se


desea llevar un bloque de
N hacia
arriba, sobre el plano inclinado, con
movimiento uniforme. Si el coeficiente
de friccin cintico entre el bloque y el
plano es 0,5. Determine la magnitud
de dicha fuerza (en N) g 10 m/s2

SEMINARIO N 02

37. La escalera de 5 m de longitud tiene


una masa de 20 kg, un nio de 40 kg
sube por ella, si solo hay friccin en la
superficie horizontal s 0,5 y k 0, 4 .
Determine la altura h que puede subir
sin que el conjunto deslice
(en m).
B

F
h
A

53

A) 175
D) 250

B) 200
E) 275

C) 225

35. En el sistema mostrado en la figura las


partculas tiene masas iguales y estn
en equilibrio. Halle el valor mnimo de
u para que no se deslicen. m1 m2

A) 0,2
D) 0,7

B) 0,3
E) 0,8

20
30
40
45
60

C) 3

38. La barra de 8 kg mostrada en la figura,


se apoya en un plano inclinado liso
(punto A) y la articulacin (punto B). El
coeficiente de friccin en el plano
horizontal es s 0,6 , k 0,5 . Si el
sistema est a punto de deslizarse,
determine la reaccin de la superficie
sobre la barra en el punto A (en N)
considere la pesa de 1 kg pequea.
A
B

liso

CEPRE-UNI

B) 2
E) 5

C) 0,5

36. Una escalera de longitud L y


N
de peso, descansa en reposo como se
muestra en la figura. Halle la friccin
(en N) con el piso 53 .
A)
B)
C)
D)
E)

A) 1
D) 4

53

s 0

1 kg

A) 40
D) 90

37

B) 44
E) 100

C) 50

39. Si la grfica representa la aceleracin


que adquiere un bloque al aplicrsele
una fuerza, seale verdadero (V) o
falso (F):
I. El grfico es una consecuencia de
la 2 ley de Newton.
II. m1 m2 .
III. m2 m1

FSICA

CICLO PREUNIVERSITARIO ADMISIN 2007-II

m2

A) 0,2
D) 3,2
m1

A) VVV
D) VFF

B) VVF
E) FFF

C) VFV

40. Un cuerpo de
kg de masa se
desplaza
sobre
una
superficie
horizontal rugosa ( k 0,2 ) bajo la
accin de una fuerza F paralela a la
superficie. Si la velocidad del cuerpo
vara con el tiempo de acuerdo a la
grfica que se muestra, determine la
magnitud (en N) de la fuerza F.
g 10 m/s2
v (m/s)
4
2
0

A) 12,5
D) 25,0

B) 15,0
E) 27,5

SEMINARIO N 02

t (s)

C) 20,0

41. Si el bloque C es abandonado en la


posicin mostrada. Determine la
aceleracin que adquiere el bloque A
(en m/s2). Desprecie el rozamiento en
todas las superficies en contacto.
g 10 m/s2 , si
mA 3mC 6mB 12m

B) 1,2
E) 4,2

C) 2,2

42. Se observa que cuando se aplica una


N a un bloque de
fuerza de F
10 kg su aceleracin resulta ser la
tercera parte de la que tiene cuando la
fuerza que se aplica es F 80 N .
Determine el coeficiente de friccin
cintica.
F

A) 0,20
D) 0,35

B) 0,25
E) 0,40

43. Indique las afirmaciones correctas


respecto de una partcula con
movimiento circular.
I. Si mantiene su velocidad angular
constante la fuerza resultante sobre
la partcula es nula.
II. Una partcula con aceleracin
angular constante cuyo radio de
trayectoria es R experimenta una
fuerza tangencial nula.
III. La velocidad lineal de la partcula
cambia de direccin debido a la
fuerza centrpeta.
A) Solo I
B) Solo II
C) Solo III
D) Solo I y II E) Solo II y III
44. Cul es el DCL que mejor representa
a la partcula en el punto A.
liso
A

r
g

A
B

C) 0,30

r
g
C
CEPRE-UNI

FSICA

CICLO PREUNIVERSITARIO ADMISIN 2007-II

A)

B)

Fc

Fc

W
D)

C)

N W

E)

Fc

B
A
cuerda

A) FFV
D) VFV

r
g

37

A) 15,8 ; no
C) 39,5 ; no
E) 20,7 ; si

B) 39,5 ; si
D) 15,2 ; no

48. Una partcula de masa m 2 kg


describe un M.C.U.V con una
trayectoria horizontal de radio R 1 m .
En el punto A de la trayectoria la fuerza
r
sobre la partcula es F 8 2 $
iN ,
calcule aproximadamente la velocidad
en el punto B de la trayectoria.
A

B) VVV
E) FFF

C) VVF

46. Sobre una pista circular de 450 m de


radio, un auto de 1000 kg acelera
desde el reposo hasta adquirir luego
de 9 s una rapidez de 30 m/s. Halle la
relacin entre los componentes
centrpeta y tangencial de la fuerza
que el piso ejerce sobre el auto.
A) 0,2
B) 0,4
C) 0,6
D) 0,8
E) 1,0
CEPRE-UNI

47. Un objeto de
kg se mueve en una
trayectoria circular en una cuerda de
0,5 m de largo. Si se mantiene una
rapidez constante de 4 m/s, cul es la
tensin en N en la cuerda cuando el
objeto est en la posicin mostrada en
la figura? Es el movimiento del tipo
MCUV?

45. La figura muestra una partcula de


masa m, atada a un hilo, que realiza
un movimiento circular en un plano
vertical. Seale la veracidad o
falsedad (F) de las siguientes
proposiciones:
I. En A la fuerza centrpeta es mayor
que la tensin.
II. En B la mnima fuerza centrpeta es
la tensin.
III. En C la fuerza centrpeta es menor
que la tensin.

r
g

SEMINARIO N 02

45

45
x

A) 2
D) 8

B) 4
E) 10

FSICA

C) 6

10

CICLO PREUNIVERSITARIO ADMISIN 2007-II

49. Un piloto desciende en picada con su


avin y cuando su velocidad es de
700 km/h describe una trayectoria
semicircular en un plano vertical,
manteniendo su rapidez constante. Si
sabe que puede soportar en el punto
ms bajo de la trayectoria un peso
aparente de hasta 6 veces su peso, el
menor radio posible de esta trayectoria
semicircular debe medir (en m)
aproximadamente:
g 9,8 m/s2
A) 756
B) 759
C) 763
D) 768
E) 771

SEMINARIO N 02

III. Un cuerpo solo puede estar en


reposo o con MRU, respecto a un
sistema de referencia inercial.
A) VVV
B) FFF
C) VFV
D) VVF
E) FVF
52. Un vagn con aceleracin constante
a se desplaza hacia la derecha, en
su interior un pasajero suelta una
moneda. Qu alternativa representa
mejor la trayectoria de la moneda
respecto al pasajero?
r
rb
a

50. Sobre
un
riel
en
forma
de
semicircunferencia de
m de radio
que se encuentra en un plano vertical
puede deslizarse una partcula de
masa m. Hasta que altura h (en m)
subir la masa cuando el riel gira con
una velocidad angular de 5 rad/s, tal
como se muestra en la figura.
A)
= 5 rad/s
superficie
lisa

A) 0,2
D) 1,6

B) 0,5
E) 1,9

C) 1,2

51. Respecto a los sistemas de referencia


inerciales, indique la veracidad (V) o
falsedad (F) de las proposiciones
siguientes:
I. Un sistema de referencia es
inercial, solo si est en reposo
respecto a Tierra.
II. Las leyes de Newton solo se
cumplen en los sistemas de
referencia inerciales.
CEPRE-UNI

B)

C)

D)

E)

53. Dadas las siguientes proposiciones:


I. En un sistema de referencia inercial
todos los cuerpos sometidos a
resultante de fuerzas cero o se
encuentran
en
reposo
o
movindose
con
velocidad
constante.
II. Un sistema de referencia inercial es
aquel que cumple con la 1ra Ley de
Newton.
III. Desde un mvil que se encuentra
en Tierra, con velocidad constante,
respecto a un observador se
observa un objeto que se mueve
con velocidad constante, entonces
sobre dicho objeto la suma de
fuerzas es cero.
Cules son verdaderas?
A) Solo I
B) Solo III
C) Todas
D) I y II
E) II y III

FSICA

11

CICLO PREUNIVERSITARIO ADMISIN 2007-II

54. Con relacin al concepto de sistemas


de referencia inerciales (SRI), seale
la veracidad (V) o falsedad (F) de las
siguientes proposiciones:
I. La aceleracin de un cuerpo es la
misma en cualquier SRI.
II. La velocidad de un cuerpo es la
misma en cualquier SRI.
III. Si un cuerpo no est acelerado
respecto
a un sistema de
referencia, entonces dicho sistema
es un SRI.
A) FFF
B) VFV
C) VFF
D) FVV
E) FVF
55. Dos observadores A y B uno fijo a
Tierra (A) y otro queur se mueve con
velocidad constante V (B), observan
que una esferita C es abandonada,
con relacin
a las siguientes
proposiciones indique verdadero (V) o
falso (F).
I. Solo A es considerado un sistema
de referencia inercial.
II. Los dos observadores medirn el
mismo tiempo de cada de C.
III. La trayectoria de C observada por
ambos sistemas (A) y (B) son
rectilneas.
B
A

ur
V

Tierra

A) FVF
D) FVV

B) VVF
E) VFF

C) FFF

56. El grfico muestra un satlite AB en


rbita alrededor de un planeta; si el
satlite recorre el tramo AB en
das y barre un rea que es el 30% del
rea COD; calcule el periodo (en das)
del satlite.
CEPRE-UNI

SEMINARIO N 02

D
C

o
A

A) 10
D) 20

B) 15
E) 25

C) 30

57. El dibujo muestra un planeta con los


satlites A y B. Si el periodo del satlite
A es de
h, calcule el periodo de B.
Asuma que RB 1,5 R A .
RB
A

RA
B

A) 32,8
D) 49,4

B) 43,2
E) 53,2

C) 45,9

58. Un satlite artificial m es puesto en


rbita alrededor de la Tierra con un
periodo de rbita de 2,8 das, un radio
de rbita 5a (en km). Determine
aproximadamente el periodo (en das)
de otro satlite artificial n cuyo radio
de rbita es 4a (en km).
A) 0,5
B) 2
C) 3
D) 4
E) 5
59. El cometa Halley se acerca al Sol a
una distancia aproximada de 0,57 UA,
y su periodo orbital es de
aos.
Qu tan lejos del sol viajar el
cometa Halley antes de que inicie su
viaje de regreso? (UA: unidad
astronmica 1UA 1,5 108 km es la
distancia media Tierra-Sol).
FSICA

12

CICLO PREUNIVERSITARIO ADMISIN 2007-II

Msol 1,99 1030 kg . Dar la respuesta


en U.A.
A) 5
B) 12
C) 17
D) 21
E) 35

SEMINARIO N 02

62. Sea F la fuerza gravitacional entre los


planetas M y 3M. Halle la magnitud de
la fuerza gravitacional sobre el cometa
m, en la posicin mostrada, debido a
M y 3M.

60. Dos satlites artificial y natural orbitan


alrededor de un planeta, tal como se
m
muestra,
determine
el
mximo
alejamiento del satlite (2) respecto del
planeta, si su periodo es 5 5 veces el
periodo del satlite (1).
60
R

(1)

(2)

3M

60
M

A)

PLANETA

C)
A) 5R
D) 10R

B) 7R
E) 15R

C) 9R

61. Con respecto a las siguientes


proposiciones, seale (V) o falso (F):
T2
I. La relacin
cte se cumple
R3
para cualquier par de planetas
independientemente del sistema al
cual pertenezcan (la cte es la
misma).
II. La 3 ley de Kepler (ley del
cuadrado
del
periodo)
se
demuestra aplicando la 2 ley de
Newton.
III. El radio medio usado en la
aproximacin
circular
de
la
trayectoria elptica de un planeta,
es la semisuma de los semiejes
mayor y menor de la elipse.
A) VVV
D) FVF

CEPRE-UNI

B) VFV
E) FFF

C) VVF

E)

m
F
M
M
13 F
m

m
F
M
1 m
D) F
3 M

13

B)

13 m
F
3 M

63. Halle el valor de la masa m ' para que


la fuerza de gravitacin resultante
sobre la masa m, ms prxima a m ' ,
sea cero. El polgono es un hexgono
regular.
m

m' ?

a
m

A) m/4
D) m

B) m/2
E) 5m/4

C) 3m/4

64. Los radios de las rbitas circulares


alrededor del Sol de Saturno y Urano
son 1,43 1012 m y 2,87 1012 m
respectivamente. Encuentre la relacin
entre las aceleraciones de los
planetas.
A) 4
B) 8
C) 16
D) 24
E) 32
FSICA

13

CICLO PREUNIVERSITARIO ADMISIN 2007-II

65. Para el sistema Sol-Tierra-Luna,


determine el cociente entre las
magnitudes de las fuerzas mxima y
mnima, actuantes sobre la Tierra, en
MSol
x
trminos
de
y
de
MLuna
RSol Tierra
y.
Suponga
rbitas
RTierra Luna
circulares.
x y2
x y2
A)
B)
x y2
x y2
x2 y
x2 y
xy
E)
xy
C)

D)

x2 y
x2 y

66. Un cuerpo pesa


N sobre la
superficie de un supuesto planeta de
radio 4100 km. Determine a que altura
(en km) sobre la superficie su peso
ser 3,96% menor.
A) 600
B) 500
C) 400
D) 350
E) 83,7
67. Determine a que altura (en km) sobre
la superficie de la Tierra un cuerpo
tendr la mitad de su peso (considere
RT 6400 km ).
A) 1381
B) 2105
C) 2624
D) 2942
E) 3557
68. A qu altura sobre la superficie
terrestre
aproximadamente
la
aceleracin de la gravedad es el 90%
de su valor al nivel del mar? (R, radio
de la Tierra)
A) R/10
B) R/20
C) R/30
D) R/40
E) R/50
69. Se sabe que g es la aceleracin de la
gravedad en la superficie de un
planeta. Determine el valor de la
aceleracin de la gravedad a una
CEPRE-UNI

SEMINARIO N 02

altura igual a 2,5R donde R es el radio


del planeta.
A) g/49
B) 4g/49
C) 5g/49
D) 6g/49
E) 7g/49
70. A qu altura sobre la superficie (en
km) de la Tierra la aceleracin de la
gravedad se reduce a la mitad del
valor en la superficie? En qu
porcentaje se reduce g a
km de
altura sobre la superficie terrestre?
g 9,8 m/s2 en la superficie de la
Tierra, ( MT 6 1024 kg, RT 6400 km
A) 9030 ; 2%
B) 2630 ; 4%
C) 15500 ; 2%
D) 2630 ; 8%
E) 9030 ; 4%
71. A qu altura sobre la superficie de la
Tierra (en trminos de su radio R) el
valor de la aceleracin de la gravedad
es la cuarta parte del valor que le
corresponde en su superficie?
R
A)
B) R
C) 2R
2
5
D) R
E) 3R
2
72. Si el bloque de
kg sube 4 metros
con velocidad constante, halle el
trabajo de la fuerza F (en J) g 10
m/s2.
F

A) 200
D) 400

B) 200
E) 800

FSICA

C) 100

14

CICLO PREUNIVERSITARIO ADMISIN 2007-II

SEMINARIO N 02

73. El sistema mostrado empieza a


moverse a partir del reposo. Halle el
trabajo que realiza la tensin de la
cuerda, para desplazar al bloque de
5 kg una longitud de 7 m.
g 10 m/s2
5 kg

A)

liso

r
g

B) 200
E) 800

C) 100

kg)
74. Sobre
el
objeto (m
mostrado en la figura, acta una
fuerza F 2i$N . Si el objeto se
desplaza 3i$m , seale la veracidad (V)
o falsedad (F) de las siguientes
proposiciones:
I. El trabajo hecho por F es 6 J.
II. El trabajo hecho por la fuerza
resultante es 6 J.
III. El trabajo hecho por la fuerza de
friccin es 1 J.
m

A) VVV
D) FVV

k = 0,1

B) VFV
E) FFF

C) VFF

F(x)

CEPRE-UNIO
F

F0d
2

76. En el extremo inferior de un resorte de


20 cm de longitud natural se cuelga un
bloque de 5 kg, el resorte se estira
5 cm. Luego, muy lentamente, se
aplica al bloque una fuerza F vertical,
que lo hace descender 10 cm. Calcule
el trabajo (en J) realizado por la fuerza.
g 10 m/s2
A) 5
B) 8
C) 10
D) 12
E) 15
77. En la figura se muestra un bloque
sobre una superficie horizontal rugosa
inicialmente en reposo, si al bloque se
aplica una fuerza variable cuyo mdulo
es F 10x 20 , x (en m) y F (en N)
considerando k 0,5 , determine el
trabajo de la friccin (en J) desde
x 0 hasta x 6m , considere la masa
del bloque 3 kg. g 10 m/s2
F

0,5

37

A) 90
D) 180

B) 120
E) 240

C) 150

78. Si se desprecia todo tipo de


rozamiento, qu trabajo (en J) realiza
la fuerza no conservativa F vertical que

X
d/2

D)

75. Sobre el bloque de masa m mostrado


en la figura, acta una fuerza F(x)
paralela al piso de magnitud variable.
Halle el trabajo de F(x) desde x 0
hasta x d .

O F
F0

C) 0

B) F0d

E) F0d

2 kg

A) 100
D) 400

F0d
2

FSICA

15

CICLO PREUNIVERSITARIO ADMISIN 2007-II

levanta la cadena de
kg de masa y
cuya longitud es 4 m, inicialmente en
reposo como se muestra en la figura,
cuando es desplazado hasta el
instante en que el ltimo eslabn
abandona la superficie con una
velocidad de 10 m/s? g 10 m/s2
F

SEMINARIO N 02

82. La figura muestra un bloque de


madera de longitud L, una bala lo
impacta con una velocidad inicial V 0 y
sale de mismo con una velocidad
v
v 0 . Cul debe ser la longitud de
2
otro bloque, del mismo material que el
anterior, para que al impactarle una
bala con velocidad igual a v0 se
detenga justo al salir?
L

v0

A) 1800
D) 700

B) 1200
E) 450

C) 500

79. Halle el trabajo neto (en J) necesario


para trasladar un cuerpo de
g de
masa, de un punto (A) a otro (B)
distantes entre si 6 m en un tiempo de
6 s. El cuerpo parte del reposo y el
movimiento
es
rectilneo
uniformemente variado.
A) 6,0
B) 106
C) 6 106
D) 0,6
E) 60
80. Un bote se est desplazando con una
energa cintica K. Qu trabajo debe
realizar el viento sobre las velas del
bote para que este duplique su
velocidad?
A) K
B) 2K
C) 3K
D) 4K
E) 5K
81. Un bloque de
kg, en reposo en el
origen de coordenadas, inicia su
movimiento bajo la accin de una
fuerza F 3x (en newton). Cul es la
variacin (en J) en su energa cintica
cuando se desplaza desde x 1 hasta
x 3m ?
A) 4
D) 16

CEPRE-UNI

B) 8
E) 20

C) 12

L
3
3
D) L
2
A)

B) L

C)

4
L
3

E) 2L

83. Una partcula de 4 kg se somete a una


fuerza que vara con la posicin, como
se muestra en la figura. La partcula
r
parte con una velocidad v 2i$m/s.
Cul es la velocidad (en m/s) en (a)
5 m y en (b) 15 m?
Fx(N)
6

A) 1,7 ; 2,9
C) 2,2 ; 3,7
E) 4,5 ; 6,3

10

15

x(m)

B) 2,9 ; 4,5
D) 3,4 ; 5,8

84. Un cuerpo parte del reposo y se


mueve sobre una superficie horizontal
por efecto de una fuerza de magnitud
FSICA

16

CICLO PREUNIVERSITARIO ADMISIN 2007-II

variable que acta siempre en la


misma direccin y sentido, tal como se
muestra en la figura. Si la masa del
cuerpo es de
kg. Cul es su
rapidez (en m/s) cuando x 1,2 m ?
F
m

37

F(N)

SEMINARIO N 02

equilibrio una distancia A y en


seguida lo soltamos, de modo que el
bloque se desliza sobre el piso
horizontal liso; entonces es correcto
que:
I. Durante el movimiento del bloque
las fuerzas que realizan trabajo
sobre el son conservativas.
II. La energa cintica es mxima en la
posicin de equilibrio.
III. La grfica energa mecnica versus
posicin es:
EM

80
A
0

A) 0,94
D) 2,77

0,6

1,2

B) 1,82
E) 3,01

x(m)

A) 0
D) 10

B) 20
E) 20

A) Solo I
D) I y III

B) Todas
E) II y III

C) I y II

87. Identifique la veracidad (V) o falsedad


(F) de las siguientes proposiciones:
r
I. Si el trabajo de una fuerza F por 2
caminos C1 y C2 diferentes, entre 2
puntos fijos, r es el mismo, se
concluye que F es conservativa.
II. La energa potencial gravitatoria
requiere de un nivel horizontal de
referencia.
III. La energa potencial elstica de un
resorte de constante k, deformado
1
x A , es kA 2 .
2

A) VVV
D) VVF

B) FVV
E) FFF

C) FVF

C) 10

86. Un pequeo bloque est conectado al


extremo de un resorte de masa
despreciable,
lo
separamos
ligeramente de su posicin de
CEPRE-UNI

C) 2,03

85. Una partcula puede desplazarse a lo


largo de cualquiera de las trayectorias
que se muestra en la figura. La fuerza
actuante sobre la partcula es
conservativa a lo largo de todas las
trayectorias excepto a lo largo de la
trayectoria CE. Si los trabajos
ejecutados
por
la fuerza
son
WAD WDE WBC 0 ,
WAB 10 J
y
WCE 20 J ; halle (en J) WCEDC .
B

+A

88. Un cuerpo de masa m se desliza sin


friccin sobre un plano inclinado, si se
suelta desde una altura h0 respecto al
piso y considerando que h es la
FSICA

17

CICLO PREUNIVERSITARIO ADMISIN 2007-II

SEMINARIO N 02

posicin del bloque en su descenso


respecto al piso en un tiempo t 0 ,
indique cul de los grficos
mostrados representa mejor el trabajo
que realiza el peso del cuerpo en
funcin de h?
W(J)

A) 180
D) 720

W(J)

h(m)

h(m)

A)

B)

W(J)
W(J)

B) 360
E) 900

C) 540

91. El bloque de
g comprime al
resorte k 100 N/m y luego se
suelta. Calcule cul debe ser la
mnima longitud que debe comprimirse
el resorte (en mm) de tal manera que
el
bloque
pueda
recorrer
la
circunferencia que se muestra en la
figura.

h(m)

C)

D)

h(m)

W(J)

A) 70
D) 120
h(m)

E)
89. A un trineo de
kg que se encuentra
sobre un lago congelado se le imparte
una rapidez inicial de 5 m/s. Si el
coeficiente de friccin cintica entre el
trineo y el hielo es 0,1; cul es la
longitud que recorre el trineo, hasta
detenerse?
A) 12,5
B) 7,5
C) 5,5
D) 2,5
E) 1,15
90. El muchacho de la figura tira de la
cuerda con una fuerza cuya magnitud
se incrementa desde cero hasta que l
quede suspendido. Si su masa es de
60 kg y k 500 N/m , calcule el trabajo
realizado sobre el resorte. (en J)

B) 80
E) 160

C) 100

92. Una esferita se encuentra suspendida


del extremo libre de un hilo muy largo.
Qu
rapidez
(en
m/s)
debe
proporcionarse a la esferita para que
ascienda hasta 1,8 metros?
A) 2
B) 3
C) 4
D) 5
E) 6
93. En el sistema mostrado en la figura
M
kg , m 5 kg , h 3 m y la
polea es ideal. Si el sistema parte del
reposo, emplee el principio de la
conservacin de la energa para
encontrar la velocidad con que el
cuerpo M llegar al piso.

M
h
FSICA

CEPRE-UNI
m

18

CICLO PREUNIVERSITARIO ADMISIN 2007-II

A) 6
D) 12

B) 8
E) 14

SEMINARIO N 02

C) 6 2

94. Un bloque de masa m 2kg parte del


reposo
y
realiza
MRUV
con
2
a 0,5 m/s . Para un tiempo de
s
halle la potencia media (en W)
compare con el promedio de las
potencias instantneas, inicial y final,
en dicho intervalo
A) 1,5
B) 2,0
C) 2,5
D) 3,0
E) 3,5
95. Se lanza en A un bloque de 2 kg con
una rapidez inicial de 12 m/s llegando
a B con una rapidez de 4 m/s.
Determine
la
potencia
media
desarrollada por la friccin (en W)
entre x 0 y x 10 m .
x
10 m

A) 200 ; 219
C) 200 ; 184
E) 100 ; 184

B) 400 ; 184
D) 400 ; 219

97. Un bloque de
kg se desplaza
horizontalmente con una rapidez
constante de 100 m/s sobre una
superficie rugosa ( k 0,8 ) por accin
r
de una fuerza horizontal F . Determine
la potencia (en kW) desarrollada por la
fuerza.
A) 80
B) 64
C) 62
D) 40
E) 16
98. En un edificio se requiere elevar agua
hasta una altura de 10 m, para ello se
usa una bomba de agua cuya
eficiencia es 40%, si la bomba es
capaz de elevar 600 litros de agua en
1 minuto. Calcule la potencia de la
bomba (en kW).
A) 1
B) 2
C) 2,5
D) 6
E) 60

30

A) 9,6
D) 20,5

B) -12,5
E) -22,4

C) -18,0

96. Sobre el bloque inicialmente en reposo


se aplica una fuerza horizontal
variables segn indica la grfica.
Cunto trabajo (en J) hace la fuerza
x 0 m hasta
x 10 m ?
desde
k 0,05 . Cul es la potencia
instantnea neta (en W) desarrollada
en x 10 m
F?
30 kg

F(N)
60

CEPRE-UNI

20

FSICA
0

x(m)
10

19

You might also like